You are on page 1of 51

AL-AMEEN COLLEGE OF

LAW INTERPRETATION OF
STATUTES
(IX sem 5years B.A.LL.B & V sem 3years
LL.B.) Model Answer Paper 2019

Mrs. Yasmeen Tabassum


(vice Principal)

Q1. What is literal interpretation? Explain.


The literal rule is the primary rule which takes precedence over the others.
Under this rule the judge is asked to consider what the legislature actually says
rather than considering what it might mean. Words and phrases should be
construed by the courts in their ordinary sense and the ordinary rules of grammar
and punctuation should be applied. In order to achieve this it is up to the judge to
give words in legislation their literal meaning even if, the effect of this is to
produce what might be considered as an unjust outcome, i.e., effect should be given
irrespective of its consequences. Example- fisher v. bell, the shopkeeper had a flick
knife on his shop window with a ticket bearing words “ejector knife”. Under the
restriction of offensive weapons act 1959 sec 1(1), “it was illegal to manufacture,
sell, hire, or offer for sale or hire or lend to any other person any knife..” . The
claimant alleged that the defendant shopkeeper had contravened sec 1(1) by
offering the flick knife for sale. It was held by the court that displaying the knife
was merely on invitation to treat, not an offer for sale and thus no liability arose.
The lack of the words “exposing for sale” in the restriction of offensive weapons
act 1959 suggested that only a true offer would be prohibited by the act.
This rule can be explained under the following head:
a) Natural and grammatical meaning: the words of a statute should first be
understood in their natural, ordinary or popular sense and phrases and
sentences are to be construed according to their grammatical meaning,
unless that leads to some absurdity or object of the statute suggests the
contrary.
It there is no ambiguity and the intention of the legislature is clearly
conveyed the court should give effect to it and not try to amend it.

Ramavtar v. assistant sales tax officer, the act of sale of vegetable does
not attract sales tax. Therefore, when an assistant sales tax officer wanted to
tax the sale of betel leaf, a vendor objected on the ground that betel leaf
was a vegetable. He claimed exemption. To
advance his argument, he relied on the dictionary meaning of vegetable in
the Shorter Oxford Dictionary and text books on botany. The SC did not
accept the scientific or dictionary meaning of the word vegetable and
observed that it must be construed not in any technical sense nor from a
botanical point of view, but as understood in common parlance. It held: It
has not been defined in the Act and being a word of everyday use it must be
construed in its popular sense, meaning that sense which people conversant
with the subject matter with which the statute is dealing would attribute to it.
It is to be understood as understood in common language. In this view, betel
leaf was not a vegetable but a condiment. Not being a vegetable, it could not
enjoy exemption from sales tax.

The judgment lays down the basic principle that a vegetable is one that in
market parlance is known as such. This is the definition of vegetable for
fiscal laws, but not so for students of botany who would regard betel leaf as
a vegetable. So the definition depends upon who asks the question. For a
botany student, betel leaf is a vegetable, but not so for a tax collector.
Therefore the sale of betel leaves was liable to sales tax.
b) Exact meaning preferred to loose meaning: it is presumed that words used
in the statute are used correctly and exactly and not loosely and inexactly.
Example- the exact meaning of the word contiguous is touching and
whereas it’s loose meaning is neighboring.
Every word has a secondary meaning too. Thus court should be careful not
to mix up secondary meaning with the loose meaning. If secondary meaning
points to that meaning which statute meant then preference should be given
to it only.
c) Technical words in technical sense-
(i) Special meaning in trade, business etc- according to this word are to be
understood in their subject matter. Thus in case of legislation relating to a
particular trade, business, profession or science words having a special
meaning in that context are understood in that sense only. Example- the
term “waist” are well known terms applied to a ship and nobody would
think of their meaning to the waist of a person when used in a an act which
deals with ships.
(ii) Legal sense of words-when words acquire a technical meaning because
of their constant use by the legislature in a particular sense or because of
their authoritative construction by superior courts, they are understood in
that sense only when used in a similar context in
subsequent legislation.
Q2. What is a statute? Explain the external aids to interpretation.
A statute is the will of the legislature. It is derived from the Latin word
“statutory” meaning “to be made to sit up”. It is a formal expression in writing
of the will of the legislature. It is usually called an act of the legislature. A
statute unless it is expressed to have a duration that is temporary, never dies,
though it may be amended or repealed.
There are aids for interpretation which are not part of the statute and these are
known as external aids. These external aids are relevant only when the
language used in the statute is not clear and two meaning are possible.
1) Dictionaries: usually a word would be defined under the definition clause
of the statute and the said word will have the same meaning for that word
whenever it occurs again in the act. But when a word is not defined in the
act itself, it is permissible to refer to dictionaries to find out the common
parlance in which that word is used. But again dictionary will have
various meaning for that one word and in selecting one out of the various
meaning, regard must be given to the context in which it s used in the act,
because the expression must take color from the context in which they
appear.
If plain reading of the provision brings out what was intended and
there is no ambiguity then dictionary meaning of a word is not
considered. Ramavtar v. assistant sales tax officer, the act of sale of
vegetable does not attract sales tax. Therefore, when an assistant sales
tax officer wanted to tax the sale of betel leaf, a vendor objected on the
ground that betel leaf was a vegetable. He claimed exemption. To
advance his argument, he relied on the dictionary meaning of vegetable
in the Shorter Oxford Dictionary and text books on botany. The SC did
not accept the scientific or dictionary meaning of the word vegetable and
observed that it must be construed not in any technical sense nor from a
botanical point of view, but as understood in common parlance. It held:
It has not been defined in the Act and being a word of everyday use it
must be construed in its popular sense, meaning that sense which people
conversant with the subject matter with which the statute is dealing
would attribute to it. It is to be understood as understood in common
language. In this view, betel leaf was not a vegetable but a condiment.
Not being a vegetable, it could not enjoy exemption from sales tax.
2) Foreign decisions: in pre-constitutional period, reference to English
decisions was a common practice despite warning by the Privy Council.
This practice is continued even now to an extent. There is one
qualification attached to the assistance of foreign decisions that prime
importance is always to be given to the language of the relevant Indian
statute, the circumstances and the setting in which it is enacted and the
Indian conditions where it is to be applied and there is always an element
of risk in taking ready and hasty assistance from foreign decisions.
3) Parliamentary history: the SC does use the aid of parliamentary history
in resolving questions of construction but its used within its circumspect
limits only in resolving ambiguities.
i. Bill- debates on a bill in parliament are not admissible for
construction of the act which is ultimately enacted. The speech only
shows the subjective intent of the speaker but it cannot reflect the
inarticulate mental process lying behind the majority vote which
carried the bill. Nor it is reasonable to assume that the minds of the
legislatures were one in accord. But in Indra Sawhney vs UOI, SC
referred to Dr. Ambedkar speech in the constituent assembly for
interpreting the expression backward classes of citizens which is not
defined in the constitution.
ii. Statement of objects and reasons- it can be referred only for
understanding the background, the antecedent state of affairs, the
surrounding circumstances in relation to the state of affairs and the
evil which the statute was sought to remedy. It cannot be utilized for
the purpose of restricting and controlling the plain meaning of the
language employed by the legislature in drafting a statute and
excluding from its operation such transactions which it plainly
covers.
iii. Commissions/inquiry committees- reports of commissions or inquiry
committees preceding the introduction of a bill have also been
referred to as evidence of historical facts or of the surrounding
circumstances or of mischief or evil intended to be remedied and at
time for interpreting the act. In Sodra Devi’s Case in which income
tax enquiry report was referred and in Express Newspaper’s case, the
press commissions report was referred.
iv. Parliamentary debates- this can be used as an external aid in
interpretation but its use is limited.
v. Letters- letter written by the law minister cannot be used as an
aid. It cannot override the statutory provision.
4) Historical facts and surrounding circumstances- the court is at liberty to
look into the history of the law, the circumstances which existed and
which lead to the passing of the law, the evil which is intended to remedy.
Where it is important to ascertain ancient facts of public nature recourse
to historical works, documents and treaties may be taken.
5) Text books- text book can be used to ascertain the true meaning of an
enactment. But it’s the discretion of the courts to accept or reject the
views given in the text book. Kesavanand Bharati vs State of
Karnataka is one such case where a large number of text books were
quoted in arriving the decision.
6) Subsequent social, political and economic developments and scientific
inventions: this can be discussed under the following heading-
i. General act- a statute may be interpreted to include circumstances
or situations which were unknown or did not exist at the time of
enactment of the statute.
When a change in social condition produces a novel situation
which was in contemplation at the time when a statute is first
enacted, there can be no prior assumptions that the enactment does
not apply to the new circumstances. Development in science and
technology must be taken into consideration while interpreting an
ongoing statute.
Example- It has been held that telephone is ‘telegraph’ within the
meaning of that word in t he telegraph act 1863 and 1869 although
telephone was not invented in 1869.
ii. Constitutional acts- a constitution is intended as an enduring
instrument, not only designed to meet the needs of the day when it
is enacted but also needs of the altering condition of the future.
Thus the language of the constitution is given a liberal
construction so as to include which in its ambit the future
developments in various fields of human activity and not just
restricting the language of the existing things at the time of passing
the constitution. Constitutional provisions are required to be
understood and interpreted with an object oriented approach. It
should not be interpreted in a narrow and pedantic sense. Every
provision should be read harmoniously and purposively and
should not render another provision redundant.
The best example can be taken of article 21 of the constitution
which provides, no person shall be deprived of his life/personal
liberty except according to procedure established by the law. Now
to this article 21 a whole lot of human rights were deduced from
art 21 like right to means of livelihood, privacy, education and
speedy trial etc.

Q3. State the effect of repeal of perpetual and temporary statutes of


interpretation.
Effect of expire of temporary statutes:
When the duration of a statute is for a specified time it is temporary statute.
After a temporary statute expires it cannot be made effective by merely
amending the same. Revival of the expired statute can be done only by re-
enacting it on similar terms or expressly saying that the expired act is
herewith revived. The effect of expiry of temporary statute can be discussed
under the following sub headings-
1) Legal proceedings under expired statute- after the expiries of an act
can the legal proceedings under that act be initiated or continued, If
that act has a saving clause its effect would be similar to Sec 6 of the
General clause act which says repeal of the statute shall not affect the
legal proceeding. But if such saving clause is not present than the
proceedings started under the temporary statute would terminate as
soon as the statute expires. Thus in the absence of a saving clause no
person can be prosecuted/ convicted under the expired act and the on-
going proceedings will be terminated at once with the expiry of the
temporary act. Rayala corporation vs director of enforcement.
2) Notifications, orders, rules etc made under temporary statute- with
the expiry of temporary act, any notification, ordered or rules issued
under it will also come to an end and cannot be revived even if the
provisions of expired act is re-enacted.
3) Expiry does not make statute dead for all purpose- even without the
saving clause the expiry does not make the statute dead for all
purpose. The nature of rights and obligations under the act has to be
considered in determining whether such rights and obligations are
continuing after the expiry of the act or no. State of Orissa vs
Bhupendra Kumar, it was held that a person who has been
prosecuted and sentenced during the continuance of a temporary act
for violating its provisions cannot be released
before he serves out his sentence, even if the temporary act expires
before the expiry of full period of the sentence.
4) Repeal by temporary statute- if a temporary statute repeals an
existing statute; the question which arises is if the repealing
temporary statute expires will the repealed statute revive? This will
depend on the construction of the repealing statute.
Mode of Repeal
Express Repeal and Repeal by implication
Repeal of a statute may be express or by necessary implication. There is no reason to
restrict the meaning of the word ‘repeal’ merely to an express repeal and to exclude
the implied one.
Express repeal of a statute is usually made by stating that the earlier statute or a
particular provision therein is thereby repealed. Usually enactments repealed are
mentioned in a schedule attached to the repealing statute. Such express repeal needs no
construction of the later statute. When the object is to repeal only a portion of an Act
words 'shall be omitted' are normally used. The legislative practice in India shows that
'omission' of a provision is treated as amendment which signifies deletion of that pro-vision
and is not different from repeal.
Substitution thus combines repeal and fresh enactment. Sometimes the expression used in the
later statute runs:
a) all provisions inconsistent with the Act are repealed, or
b) all Acts and parts of Acts in conflict with the provisions of this Act are hereby
repealed, or all laws and parts of laws in conflict herewith are expressly
repealed”. Provisions of this character leave the question open as to what laws are
inconsistent and are intended to be so repealed.
This rule must not be pressed too far. If a special enactment and a subsequent
general enactment are absolutely repugnant and inconsistent with one another, the
Courts have no alternative but to declare the prior special enactment as repealed
by the subsequent general Act. In all such cases the legislative intention, rather than
grammar or letter of the enactment, is the determining factor. If the intention is
found to be to sweep away all previous orders and to establish one rule for all
belonging to a class of persons that will be sufficient to get rid of any previous
special provision.

There is no doubt that repeal by implication is just as effective as by express


words. The general rule is that when you have an Act of Parliament enacting
particular provisions and in a subsequent Act there are provisions which are
inconsistent with the provisions of the first Act both enactments cannot stand
together. The enactment in the second Act stands and repeals the enactment in the
first Act. Of course from the necessity of the case it is
an implied and not an express repeal. Where two statutes passed in the same
year appear to be repugnant, that which was passed latest must prevail. Repeal
by implication is only affected when the provisions of a later enactment are so
inconsistent with or repugnant to the provision of an earlier one that the two
cannot stand together.
A prior general Act may be affected by a subsequent particular or special
Act if the subject-matter of the particular Act prior to its enforcement was being
governed by the general provisions of the earlier Act. In such a case the
operation of the particular Act may have the effect of partially repealing the
general Act, or curtailing its operation, or adding conditions to its operation for the
particular cases.
In Municipal Board, Bareilly v. Bharat Oil Co., the State
Government had framed rules regulating the levy of octroi in general by all
municipalities. Thereafter, rules were framed by the State Government for levy of
octroi by the Bareilly municipality expressly providing that the new rules will apply
in supersession of the existing rules. It was held that there was deemed repeal of
the earlier rules in respect of Bareilly municipality.
Presumption against Repeal
There is a presumption against repeal by implication. The reason for the
presumption is that the legislature while enacting a law has a complete knowledge
of the existing laws on the subject-matter and therefore when it does not provide a
repealing provision, it gives out an intention not to repeal the existing legislation.
The burden to show that there has been repeal by implication lies on the party
asserting it. Courts can lean against implied repeal. If by any fair interpretation both
the statutes can stand together, there will be no implied repeal. If possible, implied
repeal shall be avoided.
Where two enactments are entirely affirmative and identical no question
of inconsistency can arise.
Cases of repeal by necessary implication
Hence a statute is repealed by implication in the following cases, namely
a) If its provisions are plainly repugnant to those of the
subsequent statute.
b) If the two standing together would lead to wholly absurd
consequences
c) If the entire subject-matter of the first is taken away by the
second.
Consequences of
Repeal General
Under the common law rule the consequences of repeal of a statute are very
drastic. Except as to transactions past and closed, a statute after its
repeal is as completely obliterated as if it had never been enacted. The effect is to
destroy all inchoate rights and all causes of action that may have arisen under the
repealed statute. Therefore, leaving aside the cases where proceedings were
commenced, prosecuted and brought to finality before the repeal, no proceeding
under the repealed statute can be commenced or continued after the repeal.
Another result of repeal under the common law rule is to revive the law
in force at the commencement of the repealed statute. Thus if one statute is
repealed by a second which in turn is repealed by a third, the effect is to revive the
first statute unless a contrary intention is indicated in the third statute. The confusion
resulting from all these consequences gave rise to the practice of inserting saving
clauses in
Saving of rights acquired
The effect of clauses (c) to (e) of section 6, General Clauses Act is,
speaking briefly, to prevent the obliteration of a statute in spite of its repeal to keep
intact rights acquired or accrued and liabilities incurred during its operation and
permit continuance or institution of any legal proceedings or recourse to any
remedy which may have been available before the repeal for enforcement of such
rights and liabilities. Thus, offences committed during the continuance of a statute
can now be prosecuted and punished even after its repeal^ a course which would not
have been possible under the common law rule of complete obliteration of a
repealed statute.
The distinction between what is, and what is not a right preserved by the
provisions of section 6, General Clauses Act is often one of great fineness.
What is unaffected by the repeal of a statute is a right acquired or accrued under it
and not a mere "hope or expectation of', or liberty to apply for, acquiring a right.
A distinction is drawn between a legal proceeding for enforcing a right acquired or
accrued and a legal proceeding for acquisition of a right. The former is saved
whereas the latter is not. The question whether a right was acquired or a liability
incurred under a statute before its repeal will in each case depend on the
construction of the statute and the facts of the particular case. The right of an
injured third party to recover damages against the insurers of a motor vehicle will be
an accrued right on the happening of the accident resulting in the injury and will be
enforceable against them even after repeal of the enactment creating the liability, even
though the process of quantification is not complete before the repeal.

Revival
Under the English Common Law when a repealing enactment was repealed by
another statute, the repeal of the second Act revived the former Act. But this rule
does not apply to repealing Acts passed since 1850, and now if an Act repealing a former
Act is itself repealed, the last repeal does not revive the Act before repealed unless words
are added reviving it. The present rule is the result of the statutory provisions introduced
by the Interpretation Act of 1889, but though we are not bound by the provisions of any
English Statute, we can still apply the English Common Law if it appears to be
reasonable and proper. But it may be noted that even according to the Common Law
doctrine, the repeal of the repealing enactment would not revive the original Act if the
second repealing enactment manifests an intention to the contrary.
The common law rule of revival has been abrogated by sections 6(a) and 7 of the
General Clauses Act, 1897. The result, therefore, is that if one Act is repealed by a second
which again is repealed by a third, the first Act is not revived unless the third Act makes
an express provision to that effect. As regards revival of enactments section 6(a) covers
what is more emphatically declared by section 7, but the latter is limited in operation to
enactments; whereas the former is wider in operation and will prevent the revival of many
other matters,
e.g. the revival of a void contract or of common law. When a provision is repealed by
substitution of another provision in its place and the Act making the substitution is declared
invalid, the question of revival of the original provision requires consideration of
whether the Act has been declared invalid for want of legislative competence or on other
grounds. When a temporary statute affects a repeal of an existing statute, a question arises
whether the repealed statute revives on the expiry of the repealing statute.

Q4. Explain the common law rules relating to statutes affecting the
crown or state.
The rule of common law
a) Statutes are enacted for the subjects and not for the king, thus the crown is
not bound by any statute. There is one exception to the above rule, if the
intention of the legislature is clear that, the crown has to bind by the statute,
and then he will be bound by necessary implication. In such case by giving
his assent the king agrees to be bound by it.
b) Crown can be bound by express words or necessary implication.
c) In Bacons abridgment it was stated that the crown is bound by statute
which deals with public good, advance of religion despite the king not being
expressly mentioned there. But this view id not approved by modern writers.
d) To know if the crown is bound by necessary implication the safest way is to
read the statute as a whole and to ascertain the intention of
the legislature.
Bombay province vs. Bombay municipal corp., in this case the
question was whether the provisions of the Bombay municipal act 1888,
which authorized the commissioner to carry water mains and municipal
drains through or under any land within the city. The Bombay HC
concluded that even the government land within the city was subject to the
statutory authority of the corporation to carry water mains because the
legislation cannot operate with reasonable efficiency unless the crown is
bound by necessary implication. The Privy Council however overruled the
decision and held that there is no authority which gives support to the above
interpretation.

Extent of rule
The presumption that the crown is not bound by statutes extends to the
following class of people:
a) Sovereign personally,
b) Servant/ agents of the crown directly,
c) Persons who though not strictly servants/agents of crown are
considered to be consimili casu.(these are persons who are
independent of the crown, perform to a limited degree the regal
government function such a administration of justice, the carrying of
war, making of treaties etc.(Tamlin v Hannaford)
Rule in India
The common rule was that the crown is not bound by a statute unless
named expressly or by necessary implication. This was followed in India
before and even after the constitution. But soon this rule was overruled in
State of WB v. Corporation of Calcutta, in which the court held that a
general act applies to citizens as well as to state unless it expressly or by
necessary implication exempts the state from its operation.
Q5. “Every Statute is prima facie prospective unless it is expressly or
by necessary implication made to have retrospective operation”.
Discuss.

Every Act has a date from where it starts functioning called as the
commencement clause. The commencement of an act means the day on
which the Act comes into force. Generally a central Act comes into
operation on the day it receives presidential Assent.
A state Act comes into force on the day when the assent of the governor or
the president is the first published in the official Gazette of the state
Unless an act in brought into force by legislative authority, it cannot be said
to be in operation.
Generally the whole act is brought into force on a specified date. Even
different parts of the same Act may be brought into force on different dates.
If the statute in pending for a long time with the Government without being
passed, a writ of mandamus can be filed against the Government. The court
may direct the Government to consider the question whether the statute or
the provision should be brought into force or not.
A bill doesn’t come into force, unless it takes the form of an Act. There
are two types of Operation of statutes:
1. Prospective operation
2. Retrospective operation.

Prospective operation:
Prospective means present and future.
Prospective application means that an act starts its validity from the date of
its coming into force. It will apply for all present and future events until and
unless it is repealed.
The General rule is that a Statute when enacted has a prospective
application i.e it applies to the future.
Retrospective Operation:
Retrospective means past.
Retrospective application means an act which applies to past events and
also to present events.
All statutes are prospective in application but not all statutes have
retrospective application.
The cardinal principle of construction is based upon a Latin maxim ‘Nova
Constitutio futuris forman imponere debet non praeteritis’ i.e. a new law
ought to regulate what is to follow & not the part. It is always understood
that a statute applies to facts or circumstances which occur after the
existences of the statute unless the legislature intends to apply it
retrospectively.
For a statute to apply retrospectively it has to be expressly stated by the
legislature.
There are 5 general principles of retrospective operation:
They are as follows:
a) Power to make retrospective laws
b) Statutes dealing with substantive rights.
c) Statutes dealing with procedure
d) Recent statements of the rule against retrospectively.
e) Language not always decisive.
Power to make retrospective
laws:
Power is vested with the union parliament and state legislatures. These
two have the powers to legislate a statute to the
retrospectively and prospectively.
By stating that a particular Act is retrospective in nature, the legislature
tries to bring all those events which took place prior to the commencement
of the said Act, within the scope of that Act. Usually retrospective effect in
given to curative and validating statutes i.e. to cure the defects which existed
before the pacing of that Act.
Eg :- Declaratory Acts; procedural laws such as Cr.P.C., CPC, presumption
as to abetment of suicide by a married woman u/s. 113- A of the Evidence
Act, 1872.

Statutes dealing with Substantive Rights Statutes:


Statutes dealing with Substantive Rights Statutes/ Provisions which deal
with right-in-existence cannot be applied retrospectively.
It is a cardinal principle that every statute prima facie is prospective unless
it is either expressly or impliedly marked as retrospective.
This is based upon the maxim “nova conetitutio futuris forman imponere
debet non practeritis” (a law ought to regulate what to follow, not the past).
The rule against retrospective application lies only with respect to vested
rights is not existing rights.
Statutes dealing with procedure:
The rule of retrospective operation can be applied to procedural
statutes.Ex.1:- Cr.P.C. CPC, Law of Evidence, etc. It is prevented that in
procedural statutes, no person has a vested right in the procedure. He only
has a right of prosecution prescribed for the time being. It is only an existing
right which starts from the initiation of a case and ends upon the final
verdict.
Ex.2:- Sec. 45-B of the Employees State Insurance Act, 1948.
This section empowers the employees State Ins. Corp to recover arrears of
contribution from the Employer as arrears of land revenue. Therefore this
being procedural in nature can be applied prior to the commencement of
this section.
Recent statements of the rule against retrospectively:-
Unless the parliament clearly indicates retrospective aspect, a
statute/ a provision cannot be presumed as retrospective in nature. The
intention of the legislature is very important.
Language not always decisive:-
More important to the tense cannot be given to know whether a statute
/proviso have retrospective application or not. If the section has words like if
a person has been convicted i.e. pact tense, we cannot assume that it also
speaks about prior convictions.
Similarly the words like ‘has made’, ‘has ceased’, ‘has failed’ and ‘has
become’ many specify events happening before or after the commencement
of the Act.
What is important is to find out whether such event which place falls within
the scope of the Act.

Q6. Explain the basic principles of legislation.

The principle of utility:


The principle of utility states that actions or behavior are right in so far as
they promote happiness or pleasure and wrong if they tend to produce
unhappiness or pain. Many utilitarian’s believe that pleasure and pain are
objective states and can be more or less quantified i.e., measured. The
criterias for measuring are intensity, duration, fecundity etc. Classical
utilitarians believe in the greatest amount of happiness for the greatest
number of people. The public good ought to be the object of the legislator.
Example- A few years back, Cincinnati government officials had a
community vote on whether to use the proceeds from a proposed sales tax
increase to build two new sports stadiums for the Reds and the Bengals. A
classical utilitarian would have to examine how that expenditure would
affect everyone in the community. This determination entailed calculating
beforehand the amount of pleasure and pain that the various members of the
community would experience as a result of building those stadiums; then
decide whether the benefits (pleasures) outweigh the costs (pains)? Of
course the primary beneficiaries of the larger more modern stadiums would
be the wealthy team owners, players, perhaps a few downtown restaurant
owners, parking lot owners etc. If the levy fails, at least one of the teams will
probably move to another city. That would cause pain to some members of
the community including fans, venders, parking lot attendants etc. Defenders
of the tax argued that even the retailers in the suburban malls would benefit
economically from keeping the sports teams. If
they move they will lose customers on game days. The fundamental problem
for utilitarianism is justifying the altruistic principle of self- sacrifice in
order to benefit others. Since, I do not attend the games played at the
stadium, and will not benefit from hot dog revenues, why should I contribute
to either project? Under classical utilitarianism, at least some members of
the community must sacrifice his/her own interests for the interests of others
without benefiting personally. Indeed it is often the case that what turns out
to be in the public interest conflicts with the private interests of some
individuals. How can one justify such altruism based on utilitarian
principles?
Sometimes, it is possible for an act to provide a large amount of quantifiable
pleasure for a few persons at the cost of a small amount of quantifiable pain
for everyone else. Suppose, for example, we have a small, but significant
number of homeless children that could be helped by imposing a small tax
on everyone in Cincinnati. A utilitarian would not be able to justify imposing
that tax, unless it could be shown that more people are helped than harmed.
One way to get around this would be to count not only the number of
persons that experience pleasure and pain, but also weigh the intensity,
duration, fruitfulness, and likelihood of the pleasures involved. Hence, we
might argue that if we weigh the amount of pleasure that homeless children
experience, as the result of providing them shelter, against the minimal pain
that tax payers experience, then we might rationally justify building that
shelter at public expense.
The Ascetic Principle:
Ascetic comes from a Greek word which signifies exercise/practice. The
term ascetic has been sometimes applied to monks. The practices by which
monks sought to distinguish themselves from others were called their
exercises which consisted in so many contrivances they had for tormenting
themselves. By the principle of asceticism, Bentham meant that principle
approve actions which tends to diminish happiness and disapprove actions
which augment it.
This principle is the rival of utility principle. Under this everything that
gratifies or pleases the senses is odious and criminal. They found morality
on privation and virtue upon the renouncement of our self. They approve
everything which tends to diminish enjoyment and blame everything which
tends to increase enjoyment. This principle has been followed by two classes
of men one class are philosophers
and the other are devotees.
This principle has not been applied in nay considerable degree to the
business of government.
The arbitrary principle:
Among the principles opposed to utility, the one that now seems to have most
influence in matters of government is what may be called the principle of
sympathy and antipathy.
This principle consists in approving/ blaming by sentiments, without giving
any other reasons for the decision except the decision itself. I love/I hate
such is the pivot on which this principle turns. An action is judged to be
good or bad not because it is comfortable or the contrary to the interest of
those whom it affects, but because it please or displeases him who
judges.
The object is to make our opinions triumph without the trouble of comparing
them with the opinions of other people.
The ascetic principle attacks utility in front. The principle of sympathy and
antipathy neither rejects it nor admits it; it pays no attention to it. According
to this to legislate is an affair of observation and calculation, it is a matter
of humor and imagination of taste.

Q7. Explain the principles relating to interpretation of the constitution.


The following are some of the key principles applied specially in
interpreting the provisions of the constitution

Doctrine of pith and substance

1. Doctrine of Colorable legislation

2. Principle of Ancillary powers

3. Principle of Occupied field

4. Doctrine of repugnancy

5. Principle of Territorial Nexus

6. Doctrine of prospective overruling

Doctrine of Pith and Substance: Pith means "true nature" or "essence" and
substance means the essential nature underlying a phenomenon. Thus, the
doctrine of pith and substance relates to finding out the
true nature of a statute. This doctrine is widely used when deciding
whether a state is within its rights to create a statute that involves a subject
mentioned in Union List of the Constitution. The basic idea behind this
principle is that an act or a provision created by the State is valid if the true
nature of the act or the provision is about a subject that falls in the State list.
The case of State of Maharashtra vs. F N Balsara, illustrates this principle
very nicely. In this case, the State of Maharashtra passed Bombay
Prohibition Act that prohibited the sale and storage of liquor. This affected
the business of the appellant who used to import liquor. He challenged the
act on the ground that import and export are the subjects that belong in
Union list and state is incapable of making any laws regarding it. SC
rejected this argument and held that the true nature of the act is prohibition
of alcohol in the state and this subject belongs to the State list. The court
looks at the true character and nature of the act having regard to the
purpose, scope, objective, and the effects of its provisions. Therefore, the
fact that the act superficially touches on import of alcohol does not make it
invalid.
Thus, as held in State of W Bengal vs Kesoram Industries, 2004, the
courts have to ignore the name given to the act by the legislature and must
also disregard the incidental and superficial encroachments of the act and
has to see where the impact of the legislation falls. It must then decide the
constitutionality of the act.

Principle of Incidental or Ancillary Powers: This principle is an


addition to the doctrine of Pith and Substance. What it means is that the
power to legislate on a subject also includes power to legislate on ancillary
matters that are reasonably connected to that subject. It is not always
sufficient to determine the constitutionality of an act by just looking at the
pith and substance of the act. In such cases, it has to be seen whether the
matter referred in the act is essential to give effect to the main subject of the
act. For example, power to impose tax would include the power to search
and seizure to prevent the evasion of that tax. Similarly, the power to
legislate on Land reforms includes the power to legislate on mortgage of the
land. However, power relating to banking cannot be extended to include
power relating to non-banking entities. However, if a subject is explicitly
mentioned in a State or Union list, it cannot be said to be an ancillary
matter. For example, power to tax is mentioned in specific entries in the lists
and so the power to tax cannot be claimed as ancillary to the
power relating to any other entry of the lists.
As held in the case of State of Rajasthan vs G Chawla the power to legislate
on a topic includes the power to legislate on an ancillary matter which can
be said to be reasonably included in the topic.

The underlying idea behind this principle is that the grant of power includes
everything necessary to exercise that power. However, this does not mean
that the scope of the power can be extended to any unreasonable extent.
Supreme Court has consistently cautioned against such extended
construction. For example, in R M D Charbaugwala v State of Mysore, ,
SC held that betting and gambling is a state subject as mentioned in Entry
34 of State list but it does not include power to impose taxes on betting and
gambling because it exists as a separate item as Entry 62 in the same list.
Doctrine of Colourable Legislation: This doctrine is based on the principle
that what cannot be done directly cannot be done indirectly. In other words,
if the constitution does not permit certain provision of legislation, any
provision that has the same effect but in a roundabout manner is also
unconstitutional. This doctrine is found on the wider doctrine of "fraud on
the constitution". A thing is Colorable when it seems to be one thing in the
appearance but another thing underneath. K C Gajapati Narayan Deo
vs. State of Orissa is a famous case that illustrates the applicability of
this doctrine. In this case, SC observed that the constitution has clearly
distributed the legislative powers to various bodies, which have to act within
their respective spheres. These limitations are marked by specific
legislatives entries or in some cases these limitations are imposed in the
form of fundamental rights of the constitution. Question may arise whether
while enacting any provision such limits have been transgressed or not.
Such transgression may be patent, manifest or direct. But it may also be
covert, disguised, or indirect. It is to this later class of transgression that the
doctrine of colorable legislation applies. In such case, although the
legislation purports to act within the limits of its powers, yet in substance
and in reality, it transgresses those powers. The transgression is veiled by
mere pretense or disguise. But the legislature cannot be allowed to violate
the constitutional prohibition by an indirect method. In this case, the validity
of Orissa Agricultural Income Tax (Amendment) Act 1950 was in question.
The argument was that it was not a bona fide taxation law but a colourable
legislation whose main motive was to
artificially lower the income of the intermediaries so that the state has to
pay less compensation to them under Orissa Estates Abolition Act, 1952. SC
held that it was not colourable legislation because the state was well within
its power to set the taxes, no matter how unjust it was. The state is also
empowered to adopt any method of compensation. The motive of the
legislature in enacting a law is totally irrelevant.

A contrasting case is of K T Moopil Nair v State of Kerala. In this case, the


state imposed a tax under Travancore Cochin Land Tax Act, 1955, which
was so high that it was many times the annual income that the person was
earning from the land. The SC held the act as violative of Articles 14 and
19(1)(f) in view of the fact that in the disguise of tax a person's property was
being confiscated.

Similarly, in Balaji v State of Mysore, SC held that the order reserving 68%
of the seats for students belonging to backward classes was violative of
Article 14 in disguise of making a provision under Article 15(4).

Doctrine of Eclipse: In the case of Keshavan Madhava Menon v. The


State of Bombay, the law in question was an existing law at the time when
the Constitution came into force. That existing law imposed on the exercise
of the right guaranteed to the citizens of India by article 19(1)(g) restrictions
which could not be justified as reasonable under clause (6) as it then
stood and consequently under article 13(1) that existing law became
void “to the extent of such inconsistency”.

The court said that the law became void not in toto or for all purposes or for
all times or for all persons but only “to the extent of such inconsistency”,
that is to say, to the extent it became inconsistent with the provisions of Part
III which conferred the fundamental rights on the citizens.

This reasoning was also adopted in the case of Bhikaji Narain Dhakras
and Others v. The State Of Madhya Pradesh and Another. This
case also held that “on and after the commencement of the Constitution, the
existing law, as a result of its becoming inconsistent with the provisions of
article 19(1)(g) read with clause (6) as it then
stood, could not be permitted to stand in the way of the exercise of that
fundamental right. Article 13(1) by reason of its language cannot be read as
having obliterated the entire operation of the inconsistent law or having
wiped it out altogether the statute, book. Such law existed for all past
transactions and for enforcement of rights and liabilities accrued before the
date of the Constitution. The law continued in force, even after the
commencement of the Constitution, with respect to persons who were not
citizens and could not claim the fundamental right”.

The court also said that article 13(1) had the effect of nullifying or rendering
the existing law which had become inconsistent with fundamental right as it
then stood, ineffectual, nugatory and devoid of any legal force or binding
effect, only with respect to the exercise of the fundamental right on and after
the date of the commencement of the Constitution. Finally the court said
something that we today know of as the crux of Doctrine of Eclipse.“The
true position is that the impugned law became, as it were, eclipsed, for the
time being, by the fundamental right.”

We see that such laws are not dead for all purposes. They exist for the
purposes of pre-Constitution rights and liabilities and they remain
operative, even after the commencement of the Constitution, as against non-
citizens. It is only as against the citizens that they remain in a dormant or
moribund condition.

Thus the Doctrine of Eclipse provides for the validation of Pre- Constitution
Laws that violate fundamental rights upon the premise that such laws are
not null and void ab initio but become unenforceable only to the extent of
such inconsistency with the fundamental rights. If any subsequent
amendment to the Constitution removes the inconsistency or the conflict of
the existing law with the fundamental rights, then the Eclipse vanishes and
that particular law again becomes active again.

Doctrine of Occupied Field: In the last post, we discussed the doctrine of


Pith and Substance according to which where the question arises of
determining whether a particular law relates to a particular subject
(mentioned in one list of another), the court looks into the substance of the
matter.
There is a very thin of line of difference between doctrine of Repugnancy
and Doctrine of Occupied Field. As we know that repugnance arises only if
there is an actual conflict between two legislations, one enacted by the State
Legislature and the other by Parliament, both of which were competent to
do so.

On the other hand, doctrine of Occupied Field simply refers to those


legislative entries of State List, which are expressly made ‘subject’ to a
corresponding Entry in either the Union List or the Concurrent List.
Doctrine of Occupied Field has nothing to do with the conflict of laws
between the state and the centre. It is merely concerned with the ‘existence
of legislative power’ whereas repugnance is concerned with the ‘exercise of
legislative power’ that is shown to exist. Doctrine of Occupied Field comes
into picture even before the Union Law or the State Law has commenced.
Under Article 254, as soon as a Union law receives assent of the President,
it is said to be ‘a law made by the Parliament’. Actual commencement of the
law is not important for the purpose of attracting doctrine of Occupied
Field.
Doctrine of Territorial Nexus
Territorial Nexus and the Parliament
1. Article 245 (2) of the Constitution of India makes it amply clear that ‘No law
made by Parliament shall be deemed to be invalid on the ground that it would have
extra-territorial operation’. Thus legislation cannot be questioned on the ground
that it has extra-territorial operation.
2. It is well-established that the Courts of our country must enforce the law with
the machinery available to them; and they are not entitled to question the authority
of the Legislature in making a law which is extra-territorial.

3. Extra-territorial operation does not invalidate a law. But some nexus with India
may still be necessary in some of the cases such as those involving taxation
statutes.
Territorial Nexus and the State Legislature
The Legislature of a State may make laws for the whole or any part of the State.
Now, this leaves it open to scrutiny whether a particular law is really within the
competence of the State Legislature enacting it. There are plethoras of cases that
have stated that the laws which a state is empowered to make must be for the
purpose of that State.
Thus, the Doctrine of Territorial Nexus has been applied to the States as well.
There are two conditions that have been laid down in this respect:
1. The Connection (nexus) must be real and not illusory.
2. The liability sought to be imposed must be pertinent to that connection. If the
above two conditions are satisfied, any further examination of the sufficiency
of Nexus cannot be a matter of consideration before the courts.
In various cases relating to taxation statutes, the courts have time and again stated
that it is not necessary that the sale or purchase should take place within the
Territorial Limits of the State. Broadly speaking local activities of buying or
selling carried in the State in relation to local goods would be sufficient basis to
sustain the taxing power of the State, provided of course, such activities ultimately
result in concluded sale or purchase to be taxed.
There is also a Presumption of Constitutionality that the Legislature is presumed
not to have exceeded its constitutional powers and a construction consistent with
those powers is to be put upon the laws enacted by the Legislature.
Extra-Territorial Operation
It is well-established that the Parliament is empowered to make laws with respect
to aspects or causes that occur, arise or exist, or maybe expected to do so, within
the territory of India and also with respect to extra-territorial aspects or causes
that have an impact or nexus with India.
“Such laws would fall within the meaning, purport and ambit of grant of powers of
Parliament to make laws ‘for the whole or any part of the territory of India’ and
they may not be invalidated on the ground that they require extra territorial
operation. Any laws enacted by the Parliament with respect to extra territorial
aspects or causes that have no nexus with India would beultra vires and would be
laws made for a foreign territory.”
This clearly indicates that as long as the law enacted by the Parliament has a
nexus with India, even if such laws require extra territorial operation, the laws so
enacted cannot be said to constitutionally invalid. It is only when the ‘laws enacted
by the Parliament with respect to extra territorial aspects or causes that have no
nexus with India’ that such laws ‘would be ultra vires.

Q8. Explain the rules of interpretation of Mandatory & Directory


provisions. The words used in the statutes are true manifest of legislative intent.
The language employing impressive & commanding words like ‘shall ‘or ‘must’
convey a different intention than the language employing the gentle & liberal
words like ‘May’. Both kinds of language are incomparable. The words ‘Shall
‘or ‘Must’, render the provision rigid & demands strict compliance.
They also have inherent elements of punishment for
disobedience. A contravening act is invalid. Such are called as Mandatory
provisions.
In contrast, the use of word ‘may’ impair the rigidity of the language & grants
some kind of concessions to the subject to act or not to act. Only substantial
compliance of the provisions is sufficient an act violating the provision is valid. No
penalty is attracted if the subject does not carry strict compliance. Such provisions
are called Directory provisions.
Mandatory provisions:
The words used carry extra force. It denotes an order or a dictate. When the
legislature intends to do something it should be done in a particular way, it uses
shall e.g. 1. When expression used is ‘shall be punishable’, it means that offender
cannot escape the penal consequences. No discretion is available to the court.
2. Sec 168 of Crpc the word shall is employed in this provision. Owing to of
this word the language of the provision sounds authoritative.
If under the provision it places an obligation to the subordinate police
officer who has made the investigation to compulsory report the results to
the officer of the officer in–charge of the police station. No discretion is left
to the subordinate police officer except to furnish the results of the
investigation to officer in-charge; such provisions are considered as
mandatory.
Hence where the enactment prescribes that contemplated action to be taken
without any options or direction, such provisions are mandatory provisions.
A mandatory provision has to be complied with fully & strictly. An act done
in violation of mandatory provision is invalid
Directory provisions: on the other hand Directory provisions are those in
which the word ‘may’ is employed by the legislature. This word is of mild
character. It merely denotes probability or possibility
.It shows the way but does not impose any compulsion on the person
concerned to adopt it. E.g. Sec379, of Crpc which provides for appeal
provision, here the use of word ‘May’ which ordinary leads to conclude that
it is the choice of such convicted person whether to prefer an appeal before
the Supreme Court or not . It is at the discretion of such convicted person.
There is no command to such convicted person to move the Supreme Court.
The directory provisions are those where the acting authority is vested with
discretion, choice. Unlike mandatory provision a directory provision does
not demand strict compliance. Only substantial obedience is sufficient. If an
act is done in contravention of directory provision it
may attract some penalty but the act is not invalid. The provision which
asks a public functionary to perform a statutory function within a specified
time such provision of the statute are directory in nature. The difference
between Mandatory & Directory provisions are seen in the decisions passed
by the Supreme Court of India.
In state (NCT of Delhi) V/S Sanjay, in this case the Supreme Court held
that a statute is Mandatory or Directory depends upon certain factors like,
intention of the legislator, language & phraseology of the legislation, &
nature, design & consequences of legislation.
In Drigraj Kaur V/S Amar Krishna Narayan Singh (Raju), it was
held here that if a provision is mandatory an act done in breach thereof will
be invalid, but if it is directory, the act will be valid although the non
compliance may give rise to some other penalty if provided by the statute.
In Ram Athur Singh Bhadauria V/S Ram Gopal Singh, the Supreme
Court observed that mandatory enactment must be obeyed or fulfilled
exactly but it is sufficient if the directory enactment be obeyed or fulfilled
substantially.

Q9. Explain the internal aids of interpretation.


various parts of the same statute pressed into service by the courts for
construing anyone of its provision, are called internal aids to construction,
such aids can be invoked under the circumstances, whenever difficulty arises as to
the meaning of the statututory provision due to the ambiguity of the words & true
intention of the legislature cannot be inferred from the language, in such context it
is required to read the statute as a whole following the principle of ex visceribus
actus’, & every part of the statute may be called in aid.
Various parts of which constitutes internal aids to construction:
1. Title
2. Preamble
3. Headings
4. Marginal notes
5. Punctuation notes
6. Illustrations
7. Proviso
8. Explanation &
9. Schedules.
Title:
Earlier was not considered as important part of statute, & was not used
for interpretation,
According to Jus SR Das, title is an important part of the statute & is used for
ascertaining its general scope & throwing light on its construction. True nature of
is to be ascertain not by title but by its substance. Two kinds of titles, short title &
long title .Acc to Justice Donovan ‘long title is a legitimate aid to construction,
where there is doubtful or ambiguous, long title may be looked into to resolve that
ambiguity or doubt but in the absence of doubt or ambiguity, the meaning of the
statutes cannot be narrowed down. Use of the title for resolving an ambiguity.
Limitations of title as Internal Aids:
1. Title has no role to play where the words are precise & bear only one
meaning.
2. Title can be called in aid only when there is an ambiguity in the
language.
3. Cannot be used to narrow down or restrict plain meaning of language.
4. Title cannot prevail over clear meaning of an enactment.
5. Nature of statute can be determined by statute &not by title.

2. Preamble: it is a part of the act & it reflects the object & scope of the statute
(gist of law). The role of preamble in interpretation cannot be curtailed or
restricted. The role of the preamble can be an aid in construing a provision when
the provision is ambiguous. It can afford useful assistance to ascertain legislative
intention, but cannot control otherwise the plain meaning of the provision.
Limitations:
1. It cannot be applied as long as the words are clear & precise, giving one
meaning.
2. It can be resorted to only when the language of the provision is
capable of alternative construction.
3. It cannot extend or limit the scope of the statute.
4. It cannot over ride the provisions of the act it can only give information
on the generality of act.
5. In case of conflict b/w the provision & preamble, the latter would fail.
6. If the provision do not accord with the preamble those provisions would
cannot be invalidated on the grounds.
7. It cannot be used to control or qualify the clear & precise language of the
enactment.
8. It cannot be regarded as source of any substantive power or any
prohibition or limitation.
3. Definition or interpretation clauses: meaning of the word to avoid uncertainty,
in a statute, definition of certain words & expression used else ware in a body of
statute are commonly be found. The object of such a definition is to avoid the
necessity of frequent repetitions in describing all the subject matter to which the
word or the expression so defined is intended to apply. The principle is that all the
statutory definitions have to be read subject to the qualification variously
expressed in the definition clauses which created them & it may be that even where
the definition is exhaustive inasmuch as the word defined is said to mean a certain
thing. It is possible for a word to have a different meaning in different section of
the act it depends upon the subject or the context.
They are two kinds of definitions they are, Restrictive or exhaustive & Inclusive or
extensive.
Restrictive or exhaustive,
Which starts with the word means, Restrictive, as it does not admit of anything
more than what has been spelt in the act.
Exhaustive, because of its comprehensive nature. E.g. India defined in sec 2(g) of
Evidence Act. Meaning of deficiency, in consumer protection Act.
In Bhuwalka Steel Industrial V/S Bombay Iron & Steel Labour Board. The
SC held that use of words means in definitions clause rules aside any meaning
other than that dependent upon plain & unambiguous language of the provision.
Inclusive or extensive definition: The definition, in which the word used includes,
is known as inclusive or extensive definitions.
It is called inclusive because it embraces certain facets of meaning of words
defined leaving other open.
It is extensive because of the possibility of extension or expansion or enlargement
of meaning of defined word. The meaning is not limited to what is provided but is
capable of wider connotations.
E.g. Court, defined u/sec3 of evidence act, it says court includes all judges &
magistrates & all persons except arbitrators, legally authorized to take evidence.
Sec2 (p) of Crpc, places includes a house, building, tent, vehicles & vessel, the
word includes widen the meaning of the expression defined.
In Oswal Fats & oils Ltd V/S Add Commissioner Bareilly dist Bareilly, in this
case it was held that the word include is generally used in interpretation clauses in
order to enlarge the meaning of the words or phrases occurring in the body of the
statute & when it is so used those words or phrases must be construed as
comprehending not only such things as they signify according to their natural
import but also those things which the
interpretation clause declares that they shall include.
Exceptions to the rules that where the language uses the word ‘Means such
definition is restrictive or exhaustive in nature &where the language employs the
word Includes in a definition is extensive in nature. Even though the words
includes is used in the definition extensive in nature yet in circumstances declared
as exhaustive.
In Hemens (valuation Officer) V/S Whitsbury Farm & Stud Ltd, in this
case sec 2(3) of Rating Act 1971 was construed . This section provided the
definition of livestock. According to this definition, livestock includes any mammal
or bird kept for production of food or wool or for the purpose of its use in the
farming of land. The word livestock was not given wide meaning although the
definition employed the word includes & as such extensive in nature. It was held
that the term livestock shall not include through bred horses not kept for the use in
farming of the land.
Definition of ‘Livestock’, When both the words are used together to define a word,
it is considered to be exhaustive in nature i.e. no further addition is possible to the
enumerations already mentioned in the definition.
In Bharath coop. Bank (Mumbai) Ltd V/S Coop Bank Employees Union,
the SC held the use of words means followed by the word includes in the
definition is clearly indicative of legislative intent to make definition
exhaustive.
Ambiguous definition, sometimes the language used in the defining section is so
vague & ambiguous that the section itself requires interpretation in such cases the
ambiguous definition has to be interpreted in the light of other provisions of the
act.
In Hotel & catering etc. Board V/S Automobile Proprietary Ltd, it was held in
this case that in case of ambiguity in the definition , the definition is not to be read
in isolation , but it must be read in the context of the phrase which it defines
realizing that function of a definition is to give precision & certainty to the word or
phrase which would be otherwise be vague & uncertain, but not to contradict it.
4. Headings: They are not passed by the legislature but are subsequently inserted
after the bill has become law. In all modern statutes generally the heading are
attached to almost each section, just preceding the provision
e.g. The heading for sec 437 of Crpc 1973 is ‘when bail may be taken in case of
non-bailable offence’.
Headings can be called in as aid while construing the section but the importance
attached to heading as internal aid to construction has been differently described
by two groups of thoughts, one group says that heading is to be regarded as
preamble to the provision following them &
providing key to interpretation of clauses arranged under it. But the other group
says that headings can only be taken when enacting words are ambiguous.
They are of two kinds one which are prefix to the section & other which are prefix
to a group or a set of sections.
In N.C Dhoundial V/S Union of India. Headings can be relied upon to clear
the doubt or ambiguity in the interpretation of the provision & to discern the
legislative intent. .
Union of India V/S national Federation of the Blind, in this case it was held
that the heading of the section or marginal notes may be relied upon to clear the
ambiguity & to disearn the legislative intent but they cannot control the meaning of
body of the section. Heading of sec 33,’reservation of posts’ in persons with
disabilies Act 1995 will not play a crucial roal when section is clear &
unambiguous.
Limitations of Headings:
1. They cannot be called in aid if words are precise, & having one meaning
Acc to Lord Upjohn, while construing the act, must read the cross headings
as well as body of the act & that will always be a useful pointer as to
intention of parliament.
2. It can be referred only when language is open to more than one
construction due to uncertain meaning of words.
3. They cannot cut down nor extend the plain meaning & scope of the words.
4. It cannot explain the clear & plain meaning of the word.
5. Heading of one section or one group of section cannot be used to
interpret another section or group of section.
5. Marginal Notes: are those notes printed on the side of the section, they are
inserted by the draftsmen & not by the legislators.
Earlier they were considered as aid to construction, but now as per majority of the
opinions they are not considered helpful.
Acc to Lord Reid, marginal notes cannot be used as aid to construction. Aside note
is a poor guide to the scope of a section for it can do more than to indicate the
main object with which the section deals.
In T.M Shivashanker Pillai V/S State of Kerala, it was held that explanatory
notes do not form the part of a rule & explanatory notes need not form the part of
the rule & explanatory notes need not be looked into at all when the rule itself is
sufficiently clear for the interpretation.
In Union of India V/S National Federation of the Blind.
It was held that the heading of a section or marginal note may be relied
upon to clear of body of the section. Heading of sec 33 ‘reservation of Posts ‘in
person with disabilities Act 1995 will not play a crucial role when section is clear
& unambiguous.
Limitation of Marginal Notes:
1. Marginal notes are not considered to be good aids of construction.
2. Only those marginal notes can be used which have been inserted by the
legislation.
3. Marginal notes cannot be resorted to when only provisions are clear & giving
only one meaning.
4. It can be used only when the language is ambiguous & when more than one
construction is possible.
5. It cannot control the real meaning of the word of the statute.
6. Marginal note of one section cannot be used to interpret another section.
6. Punctuation Marks: They are some conventional marks as periods, in writing
or printing , punctuation marks means the marks such as comma, full stop,
colon ,semi colon etc. the importance of these punctuation need no description . If
the punctuation marks are ignored it may render the language meaningless. If the
punctuation mark in the language is changed the very meaning of the same set of
words might get reversed.
In Sama Alana Abdullah V/S State of Gujarat, the Supreme Court construed
the words any secret, official code or pass word or any sketch, plan, model etc. it
was observed that there is a comma present after “password” due to which the
succeeding words are separated & detached from the proceeding words. Hence the
term secret only qualified the expression “official code or pass word” but have no
application for succeeding expression “any sketch or plan, model etc”.
In Director of Public Prosecution V/S Schildkamp, here Lord Reid agreed
that punctuations can be of some assistance in construction.
In Bihar SEE V/S Pulak Enterprises, it was held that punctuations is a minor
element in the interpretation of statutes, in cases of subordinate legislation.
Limitations of punctuations: 1. no use as an internal aid.
2. Presence of comma or its absence should be disregarded if it is contrary to
plain intention of statute.
3. It cannot expand or cut down the meaning of the words.
7. Illustration: these are appended to the section of the act in order to
explain the provision of law contained in that section.
It may be considered that purpose of illustrations is to make the meaning of the
section abundantly clear by giving examples.
Mahesh Chandra Sharma V/S Rajkumar Sharma, in this case the court held
that illustration to the section is the part of the section & helps to elucidate
the principle of the section.
Mudiliyar chatterjeeV/S International Film CO. it was observed that in
construing a section, an illustration cannot be ignored or brushed aside.
In Bengal Nagpur Railways Co Ltd V/S Ruttanji Ramji, here the court stated
that the illustration cannot have the effect of modifying the language of the section.
The illustration cannot either curtail or expand the ambit of the section which
alone forms the enactment.

Limitations of illustrations:
1. Little use for interpretation as they are not part of the statutes.
2. They cannot be resorted to for construing the provision if words are
clear.
3. It can be considered only when language is ambiguous.
4. Cannot control the plain meaning
5. Cannot be used to curtail or expand the ambit of the sec.
6. Cannot modify the language
7. Cannot control the context of the section.

8. Proviso: in some sections of the statute, after main provision is spelt clause is
added like, ‘provided that.’
E.g. Sec 23, Consumer Protection Act says “ any order arrived by an order by
the National Commission in excise of its powers conferred by sub- clause (i) of
clause(a) of sec 21 may prefer the an appeal against such order to SC within a
period of 31 days.
From the date of order, Provided that SC may entertain an appeal after the
expiry of the said period of 30 days, if it feels satisfied that there is sufficient cause
for not filing it within time. The part of the section commencing with provided is
called proviso. The principal of interpretation do not permit construction without
referring to proviso attached to it. Actually proviso & section should be construed
as a whole each portion throwing light on the other.
The nature & function of the proviso is well explained in following cases: In
CIT, Mysore V/S Indo Mercantile bank Ltd
He said that the proper function of a proviso is that it qualifies the generality of
the main enactment by providing the main exception & taking out as it were from
the main enactment, a portion which but for the proviso would fall within the main
enactment.
In Anil M.K V/S State of Kerala
It was held that the normal function of the proviso is to except something out of the
enactment or to qualify something enacted therein which but for
proviso would be within the purview of the enactment. A proviso to a section
cannot be used to import into enacting part something which is not there but where
enacting part is susceptible to several meanings, it may be controlled by proviso.
Limitations of proviso:
1. It is construed in relation to section to which it appears.
2. Proviso attached to one section or sub-sec cannot be used to qualify another
section or sub-sec, nor it can be used to accept to anything from any other
section.
In Union of India V/S Dilip Kumar Singh
Here it was held that though the proviso does not travel beyond provision to which
it is appended, Golden Rule has to be applied to read the whole sec inclusive of the
proviso in such a manner that they mutually throw light on each other & result in
Harmonious construction.
1. If something is embraced by clear words in the enactment the proviso cannot be
used to nullify the same or exclude something or include something in it by
implication.
When the main proviso is clear a proviso cannot be used to cut it down.
2. The ambit & scope of the enacting section cannot be widened or curtailed by
the proviso.
3. Proviso cannot be used to frustrate the real object of the main enactment
unless the words of the proviso are such that it is necessary effect.
4. Proviso to a sec cannot be used to import into enacting part something which is
not there.
9. Explanation: In some section explanation is appended after the statutory
provisions.
An explanation is considered to be part & parcel of the enactment. They can be
many reasons for adding an explanation eg. To explain the meaning of the words
contained in the section & to remove doubts, to include something in the ambit of
main enactment or to exclude something from it , to clarify any obscurity or
vagueness in the ,main enactment.
While construing a provision, the explanation attached to it should not be
neglected.
Function of explanation is not to expand but only to explain. An explanation must
be read in the view to harmonize & clear up ambiguity of the provisions of the
section or rule. However the explanation should be restricted to the purpose for
which it has been created & should not be stretched beyond the legitimate field.
It can neither be used for curtailing or enlarging the scope of the section . It
can neither control the plain meaning of words of the section nor can have the
effect the modifying the language of the section.

10. Schedule: in many statutes the schedule is added at the end of the statute, they
are considered as the part of the statute. They provide how claim under the act can
be enforced or as to how the powers vested by virtue of the statute are to exercised
& contained prescribed forms for working out the policy underlining the sections
of the statutes .
Purpose of statutes, it is avoid encumbering the sections with the matters of
extensive details. Much importance is not attached to the schedule, but where the
words in the statute are doubtful & uncertain in meaning, schedule may be used
for throwing light on the section to determine the meaning.

Q10. Discuss the importance of Historical Facts & later developments in


interpretation of statutes.
The state of things affairs existing at the time when a law was enacted, are called
as “Historical Facts& Surrounding Circumstances”. Even the evils which
prevailed at the time when statute was passed also fall within the ambit of the term
“Historical Facts & Surrounding Circumstances”. In fact, such evils are sought to
be reminded by the enactment.
The method of Historical interpretation is employed when the language of the
statute does not give any clue as to the intention of the legislature. In this method,
the costs consider the circumstances prevailing at the time or original enactment &
decipher the intention. This intention is given effect to. In Heydons case, it was
laid down that “for the sure & true interpretation of all the statutes in general be
they penal or beneficial restricting or enlarging the common laws, four things are
to be discussed & considered.
1. The common law before making of the Act.
2. Mischief & defect for which the common law did not provide.
3. Remedy the parliament had resolved.
4. The true reasons of the remedy.

And the office of all judges shall always make such construction as shall suppress
& advance the remedy & suppress the subtle inventions & evasions for
continuance of mischief. Hence while interpreting the statutes, it is necessary to
understand the subject matter of the statute & for this purpose the court may take
into account the historical facts which existed at the time of passing the statute.
The court is also entitled to consider the surrounding circumstances prevailing at
the time of enacting of the statute.
According to Lord Atkinson, in the construction of statutes, it is permissible to
have regard to the state of things existing at the time the statute was passed & to
the evils which it was designed to remedy.
In certain cases Historical Facts cannot be taken into consideration. While
ascertaining the supposed intention of the legislature, the court cannot travel out
of the language used in the statute.
The inference from the Historical Facts & Surrounding Circumstances cannot
control the clear language employed in the enactment itself. In case of
inconsistency between the two, it is the plain language that will prevail. The
inferences drawn from historical facts & the surrounding circumstances shall have
to give way to unambiguous language.
When it becomes necessary to determine ancient facts of the public nature, the
historical works pictures, engravings, & documents may be considered. The court
may also refer to contemporary treaties which may have induced the makers of the
law to use a particular word or phrase or expression in the enactment.
In Auckland Jute Co. Ltd V/S Tulsi Chandra Goswami, here it was observed
that for the purpose of appreciating the scope & object of an old statute & for
explaining its language which may be susceptible of different meaning it may be
useful to remember the well known historical facts which led to the enactment. it is
a settled cannon of construction that the interpreter should place himself, as far as
possible in the position of those whose words he is interpreting & the meaning of
certain words & terms used in an ancient document or a statute can be properly
explained only by reference to the circumstances existing at the time when the
statute was enacted or document was written.

Limitations:
1. The court may take into account the historical facts &
surrounding circumstances prevailing at the time of enacting a
statute. But the inference drawn for it cannot be used to defeat
the clear language employed in the enactment itself.

2. In the case of conflict, the plain language shall survive & the
inferences drawn from the historical facts & surrounding
circumstances shall fail.

Q11. Explain the principles of interpretating statutes affecting the


jurisdiction of courts.
The principles regarding statutes affecting the jurisdiction of court can be
discussed under four heading.
1. Exclusion must be explicitly expressed or clearly implied:
2. Three classes of cases.
3. Cases of breach of statutory duty.
4. Omission to exercise statutory power.

1. Exclusion must be explicitly expressed or clearly implied: The provision


excluding jurisdiction of civil courts & provision conferring jurisdiction on
authorities other than civil courts are strictly construed. A strong presumption as
to civil courts have jurisdiction to decide all questions of civil nature. Hence the
exclusion of civil courts is not to be readily inferred & such exclusion must either
be explicitly expressed or clearly implied .for courts which would otherwise have
jurisdiction in respect of subject matter concerned, ouster cannot be implied.
Ouster must be expressed.
The existence of jurisdiction in civil courts to decide questions of civil nature is the
general rule & exclusion is an exception to this rule. The civil courts are courts of
general jurisdiction of the state. On this basis only the rule is made that exclusion
of jurisdiction of civil courts is not to be readily inferred.
The criminal courts are also courts of general jurisdiction & exclusion of
jurisdiction of ordinary criminal courts can be brought about by setting up courts
of limited jurisdiction in respect of the limited field, but only if the vesting and the
exercise of that limited jurisdiction is clear & operative & there is adequate
machinery for the exercise of limited jurisdiction.
In Madhav Rao Schindya V/S Union of India. It was held that a dispute with
regard to an order of the president de-recognizing all the rulers of the Indian
States passed u/Art 366(2) was in excess in authority & beyond his powers & that
the rules were entitled to the privy purse u/Art 291 & were not barred by Art 363.
In the said art the words relating to were given a restricted meaning, it was
observed that the words provisions of this constitution relating to any such treaty
etc. meant provisions having a dominant & immediate connection with the treaty.
2. Three classes of cases: According to Judge Willes, he stated that there are
three class of cases in which liability may e established founded upon the statute.
One is where there was a liability existing at common law & that the liability of the
a statute which gives a special & a particular form of remedy different from the
remedy which existed at common law, there unless the statute contains words
which expressly or by necessary
implication exclude the common law remedy the party suing has his election to
persue either that or the statutory remedy.
The second class of cases is, where the statute gives the right to sue merely,
but provides no particular form of remedy; the party can only proceed by action at
common law.
Third class where a liability not exceeding at common law is created by a
statute which at the same time gives a special & particular remedy for enforcing it.
The remedy provided by the statute must be followed, & it is not competent to the
party to persue the course applicable to cases of second class.
A statute falling in second of three classes of cases prescribes no special
remedy & whether it creates new rights & liabilities or regulates the already
existing ones, the normal through the medium of civil court, which are of civil
jurisdiction.
The first & third of three classes of cases, it has to ascertain whether the
statute in question deals with & regulates an already existing right or liability or
whether it creates a new right or liability which has no existence apart from the
statute . If the statute of the first category, the special remedy provided therein
subject to any provision for the exclusion of ordinary remedy, will only be
construed as an alternative one.
If the statute is of third category, the remedy will be construed as executive
even though the statute makes no express provision for exclusion of ordinary
remedy. Therefore the mear fact that a statute provides for certain remedies does
not itself exclude the jurisdiction of the courts. However where the new right or
liability is created by a statute which gives a special remedy for enforcing it, the
ordinary remedy of approaching the civil court is impliedly excluded.

3. Cases of breach of statutory duty.


When an act creates an obligation & enforces the performance in a specified
manner it is taken as a general rule that performance cannot be enforced in any
other manner if an obligation is created but no mode of enforcing its performance
is ordained, the common law may in general find a mode situated to the particular
nature of the cases .
In Pasmore V/S Oswald Urban District Council, In this case the question
was as to the maintainability of an action of Mandamus for enforcing the statutory
duty of local authority under sec 15 of Public Health Act to provide sufficient
number of sewers for draining their district. Sec 299 of the act provided a remedy
for enforcing the statutory duty by a complain to the Local Govt Board &
therefore it was held that there was no remedy
outside the act.
Lord Halsburry observed, the principle that were a specific remedy is given,
it thereby deprives the person who insists upon a remedy of any other form
of remedy than that given by the statute , is on which is very familiar &
which runs through the law.
4. Omission to exercise statutory power:
The normal rule is that an omission by a public authority to exercise a statutory
power conferred for the benefit of the public does not give rise to breach of duty
surrounding in damages. An example can be taken of the case of Stovin V/S Wise,
In which the Motor accident took place at a road junction partly cause the view
was being obstructed by the earth bank adjacent to the road. Although u/sec 41 &
79 of the Highway Act 1980, the local authority had statutory power to remove the
earth bank but it took no steps in that direction. It was held that by the power &
omission to exercise it did not give rise to a claim for the damages in upon the
existence of statutory power in respect of the omission to exercise the power.

Q12 Discuss Golden Rule of Interpretation.

It is the modification of the literal rule of interpretation. The literal rule


emphasises on the literal meaning of legal words or words used in the legal
context which may often lead to ambiguity and absurdity. The golden rule tries to
avoid anomalous and absurd consequences from arising from literal
interpretation. In view of the same, the grammatical meaning of such words is
usually modified. The court is usually interested in delivering justice and in order
to foresee the consequences of their decisions the golden rule is usually applied.
This rule of interpretation aims at giving effect to the spirit of the law as the mere
mechanical and grammatical meaning may not be sufficient.
Whenever the grammatical construction cannot be given without any doubt only
then shall the golden rule of interpretation be applied bearing in mind the
consequences of the decision given. Language of the law is usually an external
manifestation of the intention of the legislature underlying the law for which the
golden rule is used. This rule of interpretation is used on the basis of discretion of
the judges on giving due consideration to the consequences of the judgment given
by them. An example of the same is S. 125 of the CrPC which deals with
maintenance given to women. The court while interpreting the term ‘wife’ included
those women who have entered into bigamy, talakshuda women and divorced
women. The court has stated that even though a woman may have relinquished her
rights on divorce, she
may claim maintenance u/s 125 as she will she be regarded as a ‘wife’ 10 years
after such divorce. Further, in the case of Chandrima Das the courts interpreted
that Article 21 shall be available to non-citizens as well as citizens.
Justice Holmes stated, “A word is not crystal, transparent and unchanged. It is the
skin of the living thought and it may vary greatly in colour and content according
to the circumstances and the time in which the word is used.”Further, in the case
of Kartar Singh v. S.O. Punjab under the rent control act while interpreting ‘the
landlord requires his land for his bona fide own use’, the courts stated that that his
own use would include the even the requirements of the landlord’s son’s use.
Hence through this rule of interpretation, it becomes evident that, the text of law
along with the context in which it is applied, must be given due consideration.

Q13. Taxing statutes to be interpreted strictly explain .

A tax is imposed for public purpose for raising general revenue of the state. A
taxing statute is to be strictly construed. It is settled law that a taxation statute in
particular has to be strictly construed and there is no equity in a taxing
provision.

Principles of strict construction,


The manner in which the Income Tax has been drafted leaves great scope for
litigation. For this purpose, principles of interpretation has to be applied . These
principles by themselves are not inflallible & would depend on the facts of each
case.
1. There is no equity in tax & the principle of strict or literal
construction applies in interpretation of taxing statutes. Hence on the
plain language of the statute , if the assessee is entitled to benefits , he
has to be granted both these benefits .
2. If there are two reasonable interpretation of taxing statutes, the one
that favours the assessee has to be accepted.

The principle of strict interpretation of taxing statute , there is no


equity about a tax , there is no presumption as to tax , nothing is to be
implied , the subject can be taxed only if the case is covered within
the provisions of the taxing statute , no
tax can be imposed by the inference or by analogy or by trying to
probe into the intention of the legislature & by considering what was
the substance of the manner . Tax relief application is a mandatory
requirement for refund purpose. it is well setteled principal that tax
exemptions are strictly against taxpayers . tax refund in the nature of
the tax exemption are resolved strictly against taxpayers.

SHORT NOTES
a) Heydon’s rule.
This is a very important rule as far as the Interpretation of Statute is
concerned. It is often referred to as the “rule in Heydon’s Case”. This very
important case reported by Lord Coke and decided by the Barons of the
Exchequer in the 16th century laid down the following rules:
That for the sure and true interpretation of all statutes in general, be
they penal or beneficial, restrictive or enlarging of the common law; four
things are to be considered -

1) What was the common law before the passing of the Act?

2) What was the mischief and defect for which the common law
did not provide?

3) What remedy the Parliament hath resolved and appointed to


cure the “disease of the Commonwealth”.

4) The true reasons for the remedy.

And then the office of all the Judges is always to make such construction
as shall suppress the mischief and advance the remedy. Before proceeding
any further, a word of warning is appropriate. Uses the exact words –
“disease of the Commonwealth” - used by Lord Coke in his report and it is
important to bear in mind that words had different meanings. It is necessary
to discover their meaning at the time of writing. From the 14th century to the
end of the 17th, the meaning of disease was lack of ease, disquiet or distress
and Commonwealth, of course, meant the Country. According to an early
case, The Longford an Act must be construed as if one were interpreting it
on the day it was passed. Thus, we ask ourselves what
the word meant on the day it was uttered, if by analogy we argue that the
same can be said of a judgment. The importance of the mischief rule in the
criminal law can best be shown by considering examples. An Act of
Parliament will state the purpose for which it was enacted. If we take the
case of Parkin v. Norman, (reserved judgment), it can be seen that the
court decided that the Public Order Act 1936 was never designed to deal
with homosexual behavior in public toilets. The long title to the Act reads:
“An Act to prohibit the wearing of uniforms in connexion with political
objects and the maintenance by private persons of associations of military or
similar characters; and to make further provision for the preservation of
public order on the occasion of public processions and meetings and in
public places.”

The purposes of the Act and the mischief rule are, therefore, closely
connected, and it is very genuine to look at the long title. Another example of
the application of the mischief rule is found in Ohison v. Hylton. The
facts, briefly, were a carpenter was on his way home from work. He
boarded a train which was crowded. Another passenger objected and
subsequently both finished up on the platform. The defendant, the carpenter,
took one of his tools of his trade, a hammer, from his briefcase and struck
the other man with it. He was charged under the Prevention of Crime Act
1953.

b)Interpretation of Penal Statutes


In a penal law if there appears to be a reasonable dubiety or ambiguity, it
shall be decided in favour of the person who would be liable to the penalisation. If
a penal provision fairly be so construed as to avoid the punishment, it must be so
interpreted. If there can be two reasonable interpretations of a penal provision, the
more lenient should be made applicable.
Punishment can be meted to one only if the plain words extension of the
meaning of the word is allowable. A penalty cannot be imposed on the basis that
the object of the statute so desired. According to Maxwell, “the prerequisite of
express language for the creation of an offence, in interpreting strictly words
setting out the elements of an offence in requiring the fulfillment to the letter of
statutory conditions precedent to the infliction of punishment; and in insisting on
the strict observance of technical provisions concerning criminal procedure and
jurisdiction.”
Unless the words of a statute clearly made an act criminal, it shall not be
construed as criminal. If there is any ambiguity in the words which set
out the elements of an act or omission declared to be an offence, so that it is
doubtful whether the act or omission falls within the statutory words, the ambiguity
will be resolved in favour of the person charged. The court will inflict punishment
on a person only when the circumstances of the case fall unambiguously fall under
the letter of the law. Legislation which deals with the jurisdiction and the
procedure relation to imposition of the penalties will be strictly construed. Where
certain procedural requirements have been laid down by a statute to be completed
in a statute dealing with punishments, the court is duty bound to see that all these
requirements have been complied with before sentencing the accused. In case of
any doubt the benefit has to go to the accused even up to the extent of acquitting
him on some technical grounds. Penal provision cannot be extended by implication
to a particular case or circumstances. The rule exhibits a preference for the liberty
of the subject and in a case of ambiguity enables the court to resolve the doubt in
favour of the subject and against the Legislature which has failed to express itself
clearly, but this rule is now-a-days of limited application. The rule was originally
evolved to mitigate the rigors of monstrous sentences of trivial offences and
although the necessity and that strictness have now vanished, the difference in
approach made to penal statute as against any other statute still persists.

c.)General Rule of interpretation


If a statute laid a mandatory duty but provided no mode for enforcing it, the
presumption in ancient days was that the person in breach of the duty could be
made liable for the offence of contempt of the statute. This rule of construction is
obsolete and now has no application to a modern statute. Clear language is now
needed to create a crime. “A penal provision must be definite”. It is a basic rule of
legal jurisprudence that than an enactment is void for vagueness if its prohibitions
are not clearly defined. Pollock, CB said: “whether there be any difference left
between a criminal statute and any other statute not creating offence, I should say
that in criminal statute you must be quite sure that the offence charged is within
the letter of the law.”
In the case of Feroze N. Dotivalaz v. P.M Wadhwani and co., this
court stated: “Generally, ordinary meaning is to be assigned to any word or
phrase used or defined in a statute. Therefore, unless there is any vagueness or
ambiguity, no occasion will arise to interpret the term in a manner which may
add something to the meaning of the word which ordinarily does not so mean
by the definition itself, more particularly, where
it is a restrictive definition. Unless there are compelling reasons to do so, meaning
of a restrictive and exhaustive definition would not be expanded or made extensive
to embrace things which are strictly not within the meaning of the word as
defined.”
In Anup Bhushan Vohra v. Registrar General, High Court of
Judicature at Calcutta, the Apex Court held that the contempt proceedings being
quasi-criminal in nature, burden and standard of proof is the same as required in
criminal cases. The charges have to be framed as per the statutory rules framed for
the purpose and proved beyond reasonable doubt keeping in mind that the alleged
contemnor is entitled to the benefit of doubt. Law does not permit imposing any
punishment in contempt proceedings on mere probabilities; equally, the court
cannot punish the alleged contemnor without any foundation merely on
conjectures and surmises. As observed above, the contempt proceeding being
quasi- criminal in nature require strict adherence to the procedure prescribed
under the rules applicable in such proceedings.
A Three-Judge Bench of this Court in the case of The Assistant
Commissioner, Assessment-II, Bangalore and Ors. v. Valliappa Textiles
Ltd. and Ors., laid down:- “Though Javali (supra) also refers to the general
principles of interpretation of statute the rule of interpretation of criminal statutes
is altogether a different cup of tea. It is not open to the court to add something to
or read something in the statute on the basis of some supposed intendment of the
statute. It is not the function of this Court to supply the casus omissus, if there be
one. As long as the presumption of innocence of the accused prevails in this
country, the benefit of any lacuna or casus omissus must be given to the accused.
The job of plugging the loopholes must strictly be left to the legislature and not
assumed by the court.
So when a statute dealing with criminal offence impinging upon the liberty
of citizens, a loophole is found, it is not for judges to cure it, for it is dangerous to
derogate from the principle that a citizen has a right to claim that howsoever his
conduct may seem to deserve punishment, he should not be convicted unless that
conduct falls fairly within definition of crime of which he is charged. The fact that
an enactment is a penal provision is in itself a reason for hesitating before
ascribing to phrases used in the meaning broader than that they would ordinarily
bear. There is all the more reason to construe strictly a drastic penal statute which
deals with crimes of aggravated nature which could not be effectively controlled
under the ordinary criminal law.
While interpreting penal statutes, it is clear that any reasoning which
is based on the substance of the transaction has to be discarded. It is the duty of
the courts to apply the purpose enshrined in the unambiguous language used by
the Legislature irrespective of the fact that the statute to be interpreted is a penal
law. The courts are not allowed to give a wider meaning when the legislature has
already provided a comprehensive provision in the statute itself.
In a very recent matter of State of Rajasthan v. Vinod Kumar the Apex
Court has observed: - “awarding punishment lesser than the minimum prescribed
under Section 376 IPC, is an exception to the general rule. Exception clause is to
be invoked only in exceptional circumstances where the conditions incorporated
in the exception clause itself exist. It is a settled legal proposition that
exception clause is always required to be strictly interpreted even if there is a
hardship to any individual. Exception is provided with the object of taking it out of
the scope of the basic law and what is included in it and what legislature desired to
be excluded. The natural presumption in law is that but for the proviso, the
enacting part of the Section would have included the subject matter of the proviso,
the enacting part should be generally given such a construction which would make
the exceptions carved out by the proviso necessary and a construction which would
make the exceptions unnecessary and redundant should be avoided. Proviso is
used to remove special cases from the general enactment and provide for them
separately. Proviso may change the very concept of the intendment of the
enactment by insisting on certain mandatory conditions to be fulfilled in order to
make the enactment workable”
In this matter the sentence of the respondents was reduced by the Hon’ble
Rajasthan High Court to a lesser punishment than that prescribed under Section
376 as mandatory unless the exception is strictly complied with. The Apex Court
observed that awarding punishment lesser than the minimum sentence of 7 years
was permissible only for adequate and special reasons. However, no such reasons
have been recorded by the court for doing so, and thus, the court failed to ensure
compliance of such mandatory requirement but awarded the punishment lesser
than the minimum prescribed under the IPC. Such an order is violative of the
mandatory requirement of law and has defeated the legislative mandate. Deciding
the case in such a casual manner reduces the criminal justice delivery system to
mockery.
Purposive Interpretation Approach
It is not necessary that courts must always favour the interpretation which is
favourable to the accused and not the prosecution but it may also chose to go for
the interpretation which is consistent with the object provided in the law. In State
of Maharashtra v. Tapas D. Neogy the expression ‘any property’ in section 102
of Cr.P.C. was interpreted to be inclusive of a ‘bank account’ and hence a police
officer who was investigating the matter was justified in seizing the same. This
principle was first explained by James, L.J. who stated: “No doubt all penal
statutes are to be construed strictly, that is to say that the court must see that the
thing charged as an offence is within the plain meaning of the word used, and must
not strain the words on any notion that there has been a slip; that there has been a
casus omissus; that the thing is so clearly within the mischief that it must have
been included if thought of.
In the case of Union of India v. Harsoli Devi, a Constitution Bench of
this court laid down: - “Before we embark upon an inquiry as to what would be the
correct interpretation of Section 28- A, we think it appropriate to bear in mind
certain basic principles of interpretation of statute. The rule stated by Tindal, CJ
In Sussex Peerage case, (1844) , still holds the field. The aforesaid rule is to the
effect: “If the words of the statute are in themselves precise and unambiguous,
then no more can be necessary than to expound those words in their natural and
ordinary sense. The words themselves do alone in such cases best declare the
intent of the lawgiver.”
It is a cardinal principle of construction of statute that when language of the
statute is plain and unambiguous, then the court must give effect to the words used
in the statute and it would not be open to the courts to adopt a hypothetical
construction on the ground that such construction is more consistent with the
alleged object and policy of the Act.
In Kirkness v. John Hudson & Co. Ltd., Lord Reid pointed out as to
what is the meaning of ‘ambiguous’ and held that – “a provision is not ambiguous
merely because it contains a word which in different context is capable of different
meanings and it would be hard to find anywhere a sentence of any length which
does not contain such a word. A provision is, in my judgment, ambiguous only if it
contains a word or phrase which in that particular context is capable of having
more than one meaning.” It is no doubt true mat if on going through the plain
meaning of the language of statutes, it leads to anomalies, injustices and
absurdities, then the court may look into the purpose for which the statute has been
brought and would try to give a meaning, which would adhere to the purpose of
the statute.
Although, the person charged has a right to say that the thing charged
although within the words, is not within the spirit of enactment. But where the
thing is brought within the words, and within the spirit, there a penal enactment is
to be construed, like any other instrument, according to the fair commonsense
meaning of the language used, and the court is not to find or make any doubt or
ambiguity in the language of the penal statute, where such a doubt or ambiguity
would clearly not be found or made in the same language in any other enactment.”
Subbarao, J., has observed: “the Act (Prevention of Corruption Act, 1947) was
brought in to purify public administration.
When the legislature used the comprehensive terminology- to achieve the
said purpose, it would be appropriate not to limit the content by construction when
particularly the spirit of the statute is in accord with the words used there.” On the
same lines Hon’ble Supreme Court had widely interpreted the Food Adulteration
Act, 1954, while expressing the strong disapproval of the narrow approach of
construction to ensure that the adulterators do not exploit the loopholes in the Act.
Similarly, such pedantic interpretation has not been given in the cases relating to
section 498A of Indian Penal Code, section 12(2) of Foreign Exchange Regulation
Act, 1947 etc. The laws which have been framed for supporting the cause of
offences against women have to be sternly implemented to set an example before
the others which may deter the prospective criminals.
d.) The Ascetic Principle:
Ascetic comes from a Greek word which signifies exercise/practice. The term
ascetic has been sometimes applied to monks. The practices by which monks
sought to distinguish themselves from others were called their exercises which
consisted in so many contrivances they had for tormenting themselves. By the
principle of asceticism, Bentham meant that principle approve actions which tends
to diminish happiness and disapprove actions which augment it.
This principle is the rival of utility principle. Under this everything that gratifies or
pleases the senses is odious and criminal. They found morality on privation and
virtue upon the renouncement of our self. They approve everything which tends to
diminish enjoyment and blame everything which tends to increase enjoyment. This
principle has been followed by two classes of men one class are philosophers and
the other are devotees. This principle has not been applied in nay
considerable degree to the business of government.
e.) Codifying statute- A codifying statute is one which codifies law. It brings all
the laws relating to one subject under one act. A codifying
statute is a statute which statutes exhaustively the whole of the law upon a
particular subject. The maker of law incorporates in the enactment both the
pre-existing statutory provisions and the common law relating to the subject.
Example: Bills of Exchange act, 1882. The main object of codifying a statute
is to bring all the laws relating to the same subject under one roof or one
block or under one Act and to present uniform, orderly and authoritative
rules on a particular subject.
Example: Hindu succession Act. It codifies the law relating to intestate succession
among ‘Hindus’. A codifying statute may be a code only with respect to a
particular branch of subject.
Example: the payment of bonus act, 1965. It only deals with profit bonus. It doesn’t
deal with other categories of bonus such as customary bonus.
Codifying statutes bring an end to the conflict of decisions. A codifying statute
does not ex clued reference to earlier case laws on the subject for the purpose of
true interpretation of the words. The reference of the previous legislations is for
the reason of removal of ambiguity. The aim of codifying Statute is to declare the
law on the subject so that the judge, by true interpretation of words decides the
meaning within the parameter of such law.
In subba rao v Commissioner of income Tax, the supreme court held that
the income tax act, 1922 is a self- contained code exhaustive with the matters dealt
with therein, and its provisions show an intention to depart from common rule law
“qui facet per alium facit per se”.
Therefore codifying statutes can also be taken as external aid to interpretation.
Codifying statutes states exhaustively the whole of the law upon a particular
subject. The judge after looking in to the codifying statute will interpret the
meaning of a given term within the parameters of such law.
f. Generalia specialibus non derogant
Where there is a special provision specifically dealing with a subject, a
general provision, howsoever widely worded must yield to the former. This
principle is expressed by the maxim Generalia specialibus non derogant.
The aforesaid rule of construction was applied by the Supreme Court In
Venkataramana Devaru vs State of Mysore. In that case the Supreme Court
applied the rule to resolve conflict between Article 25 (2)(b) and 26 (b) of
the Constitution. It was held that the right of every religious denomination
or any Section thereof to manage its own affairs in matter of religion is
subject to a law made by a State providing for social welfare and reform
or throwing open of Hindu
religious institution of a public character to all classes and sections of
Hindus.
Article 25. (1) Subject to public order, morality and health and to the
other provisions of this Part, all persons are equally entitled to freedom of
conscience and the right freely to profess, practise and propagate religion.
(2) Nothing in this article shall affect the operation of any existing law or
prevent the State from making any law (a) regulating or restricting any
economic, financial, political or other secular activity which may be
associated with religious practice;
(b) providing for social welfare and reform or the throwing open of
Hindu religious institutions of a public character to all classes and sections
ofHindus.
Article 26. Subject to public order, morality and health, every religious
denomination or any section thereof shall have the right—(a) to establish
and maintain institutions for religious and charitable purposes; (b) to
manage its own affairs in matters of religion; (c) to own and acquire
movable and immovable property; and (d) to administer such property in
accordance with law.
In State of Gujarat vs. Ramji Bhai, Supreme Court taking note of the
principle observed as follows: “Generalia Specialibus non derogant is a
cardinal principle of interpretation. It means that the general provisions
must always yield to the special provisions. Construed in accordance with
this fundamental principle, the special class of unregistered dealer covered
by Section 33 (6) must be taken to have been excluded from the purview of
the general provisions in Section
35. Thus considered, it is clear that the case of an unregistered dealer who
evades tax by committing the double default specified in Section 33(6),
action can be taken only under that Section and not under Section 35”.
Thus, it is well settled that if a special provision is made on a certain
matter, that matter is excluded from the general provision. In the event of
conflict between a general and a special provision, the latter must prevail.
Differently stated the principle is that general words in a Statute should not
be held to repeal or rip up a specific provision upon a particular matter. A
general rule though stated in wide terms must be taken to be not interfering
with matters covered by a special provision.
In South India Corporation (P) Ltd. vs. Secretary, Board of Revenue,
Trivendrum, it was held that the general provision under Article 372 of
the Constitution regarding continuance of existing laws is subject to
Article 277 of the Constitution, which is a special provision relating to
taxes, duties, cesses or fees lawfully levied at the commencement of the
Constitution. In this regard, the Supreme Court observed as follows:-
“With this background let u now consider the following two questions raised
before us: (i) whether Article 372 of the Constitution is subject to Article 277
thereof; and (ii) whether Article 372 is subject to Article 278 thereof. Article
372 is a general provision’; and Article 277 is a special provision. It is
settled law that special provision should be given effect to the extent of its
scope, leaving the general provision to control cases where the special
provision does not apply. The earlier discussion makes it abundantly clear
that the constitution gives a separate treatment to the subject of finance and
Article 277 saves the existing taxes etc. levied by states, if the conditions
mentioned therein are complied with. While Article 372 saves all pre-
Constitution valid laws, Article 277 is confined only to taxes, duties, cesses
or fees lawfully levied immediately before the Constitution. Therefore,
Article 372 cannot be construed in such a way as to enlarge the scope of the
savings of taxes, duties, cesses or fees. To state it differently, Article 372
must be read subject to Article 277. We have already held that an agreement
can be entered into between the Union and the States in terms of Article 278
abrogating or modifying the power preserved to the State under Article
277”.
In Gujarat State Co-operative Land Development Bank vs P.R.
Mankad, the Supreme Court applying the maxim generalia specialibus
non-derogant held that a general provision must yield to the special
provision.
Lord Hobhouse in Barker vs Edgar opined that when the legislature had
given its consent to a separate subject and made provision for it, the
presumption is that a subsequent general enactment is not intended to
interfere with the special provision unless it manifests that intention very
clearly.

g. Harmonious construction.
Harmony means peace. Peace is essential at times of war or internal
conflicts. In legal sphere, harmony is necessary when two or more provisions of
the same Act or two different Acts are conflicting with each other, the courts while
interpreting those provisions will try to restore peace/ harmony between them.
Harmony is restored by removing the
conflicting part and making the provisions work in-to.
When two or more provisions of the same statute are repugnant, the court
will try to construe the provisions by harmonizing them with each other. One
provision of an Act doesn’t make another provision of the same act void.
The legislature does not want itself to put in any dilemma by enacting 2
conflicting provisions in the same Act or different Acts. But as human err
sometimes the legislature makes provisions which conflict with each other. At that
time the courts apply Harmonious Construction and try to balance conflicting
sections. This is called Harmonious construction.
State of Bombay v F N Balsara, Constitutional validity of Bombay
prohibition Act. 1949 was in question (Bombay legislation entry 31 list II) said that
the state legislature has the power to prohibit possession, use and sale of
intoxicating liquor absolutely. Entry 19 list I- import sale or possession of article
imported into the country by a person residing in the territory into which it is
imported. The state legislature has encroached upon the powers of union by
enacting Bom P Act. Held: no conflict b/w entry 31 of list II and entry 19 of list I.
Bombay prohibition act does not encroach upon the field assigned to the federal
legislature.
T.M.A pai foundation v state of Karnataka: Article 29 and 30 of the
Indian constitution should be interpreted harmoniously so that object and purpose
can be achieved.
Art. 29 secure to every citizen the right to conserve its own language, script or
culture.
Art. 30 guarantee every religious or linguistic minority, the right to establish and
administer educational institutions of their choice.

PROBLEMS:

a. B, abused A, using unparliamentarily words against him. A,


angered by this hit B on his head with a rod. B fell down
unconscious & was taken to hospital, where he was declared &
brought dead. A, was challenged with the commission of a crime
under IPC. But A denied the criminal liability holding that he
exercised his right of self defense & the death was an accident.
Whether A, is guilty of crime? Decide with the appropriate rule
of interpretation.

In this case ‘A’ has committed the crime by causing grievous hurt which has
amounted to the death of ‘B’, which attracts IPC, as IPC being penal law or
statute it is to be strictly interpreted & construed. As the act
committed by ‘A’ here is a wrong against the state which falls in the second
category of strict construction of penal statutes.
The principle that a statute enacting an offence or imposing a penalty has to
be strictly construed. A penal statute enacts the offence or imposes penalty. A
person cannot be punished unless all ingredients of an offence are present. A
person cannot be punished unless all ingredients of the penal provision are
satisfied otherwise it will become violation of Art 20(1) & 21 of Indian
constitution.
Penal statutes should be construed strictly but such construction should not defeat
the purpose of the act. Some laws are enacted to cure some immediate mischief &
bring into effect some kind of social reform. Some legislation declares certain acts
wrongful & invalid & forbid them.
Wrongful acts may be categorized into two classes, firstly, wrongful act against an
individual &secondly, wrongful act against the state.
The above offence falls under the second category i.e., the commission of such
offence is deemed to be against state. The state imposes penalty on offender like
imprisonment, fine, forfeiture or any other penalty. The statutes dealing with
wrongful acts against the public are called penal statutes. The object of these
statutes is to punish the offender by imprisonment, fine or forfeiture etc. these must
be strictly interpreted, & to be construed strictly in accordance with law.

b. Prakash Ayurvedic Bhavan manufactures medical preparations for


internal & external use of human beings. For manufacturing of
medicines many ingredients are being used & liquor is one among
them. The excise commissioner sought to impose excise duty on the
quantum of liquor used for the medical preparation. This was
objected by the bhavan that it was a medicine & not alcohol that
they manufacture & sell. Decide applying suitable rule of
interpretation. Whether imposing excise duty is justifiable.

In this case the medicine prepared by the Bhavan is liable to be taxed by the excise
commissioner, the Bhavan cannot object on the grounds that it is just a medicine.
As this comes in the concept of the principles of strict construction of taxing
statutes means that the subject was bound to pay tax only when he was asked to do
so within the letter of the law & that if there were two reasonable interpretations
possible, that which helps the subject should be accepted. The word alcohol does
not have two interpretations. It has clear & unambiguous meaning. If a medicine
or a toilet preparation contains
alcohol, it is subject to duties under the act. It is therefore irrelevant whether the
medicine contains pure alcohol or some substance which in itself contains alcohol.
Hence medicines containing alcohol does are subjected to taxes under the taxing
laws.
A taxing statute should be strictly construed in favor assessee in such case only
where the expression used in the statute is capable of two reasonable
interpretations &that being not the situation in this case the Bhavan is liable for
excise duty under the act & it cannot claim for exemptions.

c) Industrial disputes act provides that a party cannot be represented by a


legal practitioner before labour court, tribunal or national tribunal.
Whereas the advocates act confers right to practice on every advocate
before any authority. How to resolve the clash?

In this case the industrial disputes act is an earlier special law and the advocates
act is a later general law. The general principal is that an earlier special act does
not impliedly repeal the later general law, unless both the provisions cannot stand
together.

In Paradip port trust vs. their workmen, the court held that the industrial dispute
act is not in conflict with the advocates act as Sec 36(4) of the industrial disputes
act states that “a party to the dispute may be represented by a legal practitioner
with the consent of the other parities to the dispute”.

From this case we can see that sec 36(4) does not bar the advocates to represent
their clients completely, it just puts a condition. Thus in the above case both the
provisions can operate in their respective fields.

d) Employer has to pay maternity benefit by way of wages for the actual
period of a woman’s absence for six weeks under maternity benefit act
while calculating wages he excludes Sundays. Meena questions it and
claims that she should get wages for Sunday also. Advice her with the
help of suitable rule of interpretation.

Beneficent construction has to be in the above case as Maternity benefit act is


beneficial or welfare legislation. Beneficial legislation should be given widest
possible interpretation within the corners of its wordings. This above case is
similar to B Shah vs. Presiding officer, labour court, in this case it was held that
sec 5 of the maternity benefit act says that the employer is liable to pay maternity
benefit to woman workers at the rate of
average daily wages for the period of her actual absence immediately preceding
and including the day of her delivery and for 6 weeks immediately following that
day. The question here was whether Sundays are to be included. The SC held that
this act being a beneficent legislation, it is enacted for the benefit of the working
mothers hence it was held Sundays are also included.

You might also like